Hardware Upgrade Forum

Hardware Upgrade Forum (https://www.hwupgrade.it/forum/index.php)
-   Scienza e tecnica (https://www.hwupgrade.it/forum/forumdisplay.php?f=91)
-   -   [Official Thread]Richieste d'aiuto in MATEMATICA: postate qui! (https://www.hwupgrade.it/forum/showthread.php?t=1221191)


kwb 17-05-2013 17:25

Qualcuno mi sa dare una piccola delucidazione sull'estensione di una funzione ( estensione, attenzione, non supporto! )?
Se ho una funzione composta del tipo:


Con:


Qual è l'estensione della funzione composta?

xxxyyy 06-06-2013 02:10

che voi sappiate, esiste un simbolo per dire "per quasi ogni".
Come scrivereste "per quasi ogni x"?

Ziosilvio 06-06-2013 07:31

Quote:

Originariamente inviato da xxxyyy (Messaggio 39560847)
che voi sappiate, esiste un simbolo per dire "per quasi ogni".
Come scrivereste "per quasi ogni x"?

Simboli logici, che io sappia, no: perché la definizione di "quasi ogni" richiede non solo la logica, ma anche la teoria della misura.
Se vuoi scrivere "P(x) per quasi ogni x" in logica simbolica, devi usare un'espressione lunga, del tipo:
Esistono, però, alcune abbreviazioni:
- "a.e." per "almost everywhere", quasi ovunque;
- "i.o." per "infinitely often", infinitamente spesso.

xxxyyy 07-06-2013 23:38

Un'altra domanda sulle notazioni.
Mettiamo di avere una funzione di 4 variabili, f(x,y,z,t). Due sono quantificate (mettiamo z e t), quindi la funzione dipende solo dalle altre due. Come si scrive la f?
Ad esempio voglio dire f 'appartenente a' C°(A), A 'sottoinsieme del piano'.
Grazie
:)

lupin 3rd 11-06-2013 20:05

scusate raga, non mi è chiaro il significato delle seguenti proprietà del valore assoluto:
|a| ≤ b se −b ≤ a ≤ b
|a| ≥ b se a ≤ −b v a ≥ b

qualcuno me le può spiegare meglio?

Ziosilvio 12-06-2013 10:01

Quote:

Originariamente inviato da lupin 3rd (Messaggio 39586257)
scusate raga, non mi è chiaro il significato delle seguenti proprietà del valore assoluto:
|a| ≤ b se −b ≤ a ≤ b
|a| ≥ b se a ≤ −b v a ≥ b

qualcuno me le può spiegare meglio?

Non ti sono chiare perché sono sbagliate, dovrebbero essere:

|a| <= |b| se e solo se -b <= a <= b
|a| >= |b| se e solo se (a <= -b oppure b <= a)

Come spiegazione veloce: il valore assoluto di un numero reale è la sua distanza dallo zero sulla retta reale.

lupin 3rd 12-06-2013 10:42

Quote:

Originariamente inviato da Ziosilvio (Messaggio 39588232)
Non ti sono chiare perché sono sbagliate, dovrebbero essere:

|a| <= |b| se e solo se -b <= a <= b
|a| >= |b| se e solo se (a <= -b oppure b <= a)

Come spiegazione veloce: il valore assoluto di un numero reale è la sua distanza dallo zero sulla retta reale.

no no, sono giuste (stamattina ho capito), le ho prese da un libro di matematica:

1) |a| ≤ b se −b ≤ a ≤ b

se a > 0 |3| ≤ 6 quindi 3 ≤ 6 quindi −6 ≤ 3 ≤ 6
se a < 0 |-3| ≤ 6 quindi 3 ≤ 6 quindi −6 ≤ -3 ≤ 6


2) |a| ≥ b se a ≤ −b v a ≥ b

se a > 0 |6| ≥ 3 quindi 6 ≥ 3
se a < 0 |-6| ≥ 3 quindi 6 ≥ 3 quindi -6 ≤ −3

Lampo89 12-06-2013 11:36

Quote:

Originariamente inviato da lupin 3rd (Messaggio 39588492)
no no, sono giuste (stamattina ho capito), le ho prese da un libro di matematica:

1) |a| ≤ b se −b ≤ a ≤ b

se a > 0 |3| ≤ 6 quindi 3 ≤ 6 quindi −6 ≤ 3 ≤ 6
se a < 0 |-3| ≤ 6 quindi 3 ≤ 6 quindi −6 ≤ -3 ≤ 6


2) |a| ≥ b se a ≤ −b v a ≥ b

se a > 0 |6| ≥ 3 quindi 6 ≥ 3
se a < 0 |-6| ≥ 3 quindi 6 ≥ 3 quindi -6 ≤ −3

Come ti ha fatto notare Ziosilvio, non sono proprio corrette le relazioni che hai postato... vedila in questi termini
dici che: |a| <= b se -b <= a <= b

prendi b negativo, tu dici che se a fosse più piccolo di b E contemporaneamente più grande di -b allora vale |a| <= b ... se prendi b = -10 qual è l'insieme degli a per cui 10 <= a <= -10 ? ha senso come scrittura?
p.s. credo che comunque sul libro sia implicito il fatto che, in quella formula, sia b positivo...

lupin 3rd 12-06-2013 13:31

Quote:

Originariamente inviato da Lampo89 (Messaggio 39588836)
Come ti ha fatto notare Ziosilvio, non sono proprio corrette le relazioni che hai postato... vedila in questi termini
dici che: |a| <= b se -b <= a <= b

prendi b negativo, tu dici che se a fosse più piccolo di b E contemporaneamente più grande di -b allora vale |a| <= b ... se prendi b = -10 qual è l'insieme degli a per cui 10 <= a <= -10 ? ha senso come scrittura?
p.s. credo che comunque sul libro sia implicito il fatto che, in quella formula, sia b positivo...

si infatti scusatemi, sul libro sta scritto b € (0, + inf)

(€=appartiene, inf =infinito)

quindi ciò che ho scritto io nel post precedente è valido solo per b > 0...ok grazie ;)

lupin 3rd 13-06-2013 14:42

se volessi determinare le soluzioni di questa equazione come si fa?

|x+2| ≤ |2x-3| + 1

Ziosilvio 14-06-2013 08:47

Quote:

Originariamente inviato da lupin 3rd (Messaggio 39595150)
se volessi determinare le soluzioni di questa equazione come si fa?

|x+2| ≤ |2x-3| + 1

Il valore assoluto non si comporta bene con le operazioni lineari, quindi devi considerare separatamente i casi in cui gli argomenti del modulo sono positivi o negativi, e in corrispondenza scrivere altrettante disuguaglianze, che dovrai verificare una per una.
Ad esempio, x + 2 e 2x - 3 sono entrambi negativi per x < -2, quindi con questa restrizione la disuguaglianza diventa - (x + 2) <= - (2x - 3) + 1: risolvi questa, e vedi se le soluzioni rispettano il vincolo x < -2 che hai dato. Poi fai lo stesso con gli altri casi.

antcos 16-06-2013 16:03

Limite con logaritmo naturale e numero di nepero
 
1 Allegato(i)
applicando de l'hopital me la ritrovo sotto e quindi il problema non viene risolto:muro:

robertogl 16-06-2013 16:11

Quote:

Originariamente inviato da antcos (Messaggio 39607494)
applicando de l'hopital me la ritrovo sotto e quindi il problema non viene risolto:muro:

io ti direi di ignorare il logaritmo, e fare il limite solo del suo argomento, poi lo rimetti dentro e vedi cosa succede :) non so però se è un approccio molto onesto :)

antcos 16-06-2013 16:19

Quote:

Originariamente inviato da robertogl (Messaggio 39607532)
io ti direi di ignorare il logaritmo, e fare il limite solo del suo argomento, poi lo rimetti dentro e vedi cosa succede :)

infatti è quello che pensavo.Ma facendo la forma indeterminata rimane:stordita:

DanieleC88 16-06-2013 16:29

Quote:

Originariamente inviato da antcos (Messaggio 39607494)
applicando de l'hopital me la ritrovo sotto e quindi il problema non viene risolto:muro:

Io inizierei intanto col notare che:

Codice:

ln(a / b) = ln(a) - ln(b)
Quindi, puoi trasformare l'argomento del limite così:

Codice:

ln(e^sqrt(x) / x) = ln(e^sqrt(x)) - ln(x)
Siccome il logaritmo naturale è proprio in base e, la prima parte si semplifica così:

Codice:

ln(e^sqrt(x)) - log(x) = sqrt(x) - ln(x)
Dividendo tutto per ln(x) hai: (e puoi farlo perché per x → +inf è diverso da zero, ovviamente)

Codice:

(sqrt(x) - ln(x))/ln(x) = sqrt(x)/ln(x) - 1
Per cui ora puoi fare il limite per x → +inf del solo sqrt(x)/ln(x).

Rimarrebbe una forma indeterminata, ma applica la regola di de l'Hôpital e ti viene:

Codice:

lim x→+inf [ sqrt(x)/ln(x) ] - 1 = lim x→+inf [ x/2sqrt(x) ] - 1 = lim x→+inf [ sqrt(x)/2 ] - 1 = +inf
E hai risolto. Forse ci sono metodi più ovvii, ma dovrebbe essere una risoluzione corretta.

antcos 16-06-2013 16:34

Quote:

Originariamente inviato da DanieleC88 (Messaggio 39607588)
E hai risolto. Forse ci sono metodi più ovvii, ma dovrebbe essere una risoluzione corretta.

grazie:mano:
PS:comunque se si potesse usare Mathjax su questo forum sarebbe meglio(io almeno mi trovo meglio con quest'ultimo)

lupin 3rd 19-06-2013 11:36

Quote:

Originariamente inviato da Ziosilvio (Messaggio 39598107)
Il valore assoluto non si comporta bene con le operazioni lineari, quindi devi considerare separatamente i casi in cui gli argomenti del modulo sono positivi o negativi, e in corrispondenza scrivere altrettante disuguaglianze, che dovrai verificare una per una.
Ad esempio, x + 2 e 2x - 3 sono entrambi negativi per x < -2, quindi con questa restrizione la disuguaglianza diventa - (x + 2) <= - (2x - 3) + 1: risolvi questa, e vedi se le soluzioni rispettano il vincolo x < -2 che hai dato. Poi fai lo stesso con gli altri casi.

anche se in ritardo, ti ringrazio :D

CioKKoBaMBuZzo 04-07-2013 21:08

ciao a tutti

ho una domanda, ma non so neanche se la domanda è giusta :D
comunque ci provo
ho una funzione f: R -> R monotona crescente

se ho l'equazione:
f(x)*x = f(y)*y

è possibile in qualche modo dire che monotonia implica x = y?

se la domanda non ha senso cercherò di capire meglio cosa voglio sapere :D

grazie

CioKKoBaMBuZzo 05-07-2013 10:17

come non detto, ho trovato la dimostrazione, era una stupidata

grazie comunque :D

stgww 21-07-2013 14:23

Esercizio geometria analitica
 
Ciao a tutti!
Non riesco a capire come svolgere questo esercizio, avete qualche suggerimento?

Sia la sfera S in con centro nell'origine e raggio 2
a)Scrivere l'equazione cartesiana del piano P passante per Q(1,1,0) e parallelo all'asse z che taglia S lungo la circonferenza T di raggio = radq(2)

b)Scrivere l'equazione del cilindro C con generatrici perpendicolari al piano P avente come direttrice T

c)Determinare l'equazione di una sfera tangente al piano P e inscritta nel cilindro C e il numero di tali sfere

Grazie

zanardi84 22-07-2013 15:34

Ho risolto un problema intuitivamente, ma in modo rigoroso non ci so arrivare.

Ho un'urna con 21 lettere dell'alfabeto. Estraendone una devo calcolare la probabilità che sia o una vocale, o una lettera che precede la M.

P1 probabilità che sia vocale = 5/21
P2 probabilità che preceda M = 10/21

I due eventi sono compatibili perchè può essere sia vocale che precedere M: A, E, I. Quindi 3/21. Questo intuitivamente.
E applico il teorema di probabilità: P1 + P2 - P1 intersez P2.

Ora: come arrivo all'intersezione con un procedimento matematico rigoroso? So che devo considerare che esiste una forma di ordinamento delle lettere.

Grazie.

zanardi84 25-07-2013 14:37

Un altro problema:
7 palline numerate da 1 a 7 in un'urna. Devo descrivere la variabile aleatoria che che definisce il maggiore dei due numeri estratti contemporaneamente.

Per quanto riguarda i valori della variabile, essi vanno dal 2 al 7 perchè in estrazione di una coppia 1 non sarà mai il valore maggiore.
Ma le probabilità dei valori della variabile come si calcolano?

zanardi84 14-08-2013 15:59

Ho un limite che non riesco a risolvere: lim per x -> più o meno infinito di

2x log(fx) dove fx = (x^2 + 3x) / (X^2 +1)

Mi potreste indicare un modo?

zanardi84 06-11-2013 17:20

Vorrei la conferma per un limite

lim per x -> 0+ di x sin(log x)

dico che: -1 <= sin(log x) <= 1

moltiplico tutto per x perciò -x <= x sin (log x) <= x

per il teorema del confronto (carabinieri), se gli etremi tendono a 0, allora anche il centrale tende a 0.

robertogl 06-11-2013 17:24

Io direi che logx va a meno infinito, ma il sin sta sempre tra meno uno e uno, quindi in realtà per qualsiasi logaritmo, se il sin è moltiplicato per zero, tutto il limite va a zero :)

Sent from my GT-I9505 using Tapatalk 4

Lampo89 06-11-2013 18:15

Quote:

Originariamente inviato da zanardi84 (Messaggio 39834677)
Ho un limite che non riesco a risolvere: lim per x -> più o meno infinito di

2x log(fx) dove fx = (x^2 + 3x) / (X^2 +1)

Mi potreste indicare un modo?

se aggiungi e togli a numeratore di f(x) il termine uno ottieni qualcosa di "illuminante"
(x^2 + 3x +1-1) / (X^2 +1) = 1 + (3x - 1)/(x^2 +1)
utile perché il secondo termine a membro dx tende a zero nel limite considerato e log(1+ eps) = eps + o(eps) dove eps è una quantità piccola.

Ziosilvio 06-11-2013 19:16

Quote:

Originariamente inviato da zanardi84 (Messaggio 40233438)
Vorrei la conferma per un limite

lim per x -> 0+ di x sin(log x)

dico che: -1 <= sin(log x) <= 1

moltiplico tutto per x perciò -x <= x sin (log x) <= x

per il teorema del confronto (carabinieri), se gli etremi tendono a 0, allora anche il centrale tende a 0.

Infatti è proprio così: infinitesimo per limitato tende a zero.

zanardi84 07-11-2013 10:02

Grazie.

Ho un altro limite che non riesco a risolvere

lim per x ->+ infinito di x*(e^f - radice di e)

dove f = (x+2)/(x+3)

Razionalizzare non serve perchè non ho variabili sotto la radice, se svolgo il limite dell'esponente ottengo una forma di indecisione perchè nella sottrazione ottengo radice di e - radice di e, cioè 0, moltiplicato per + infinito che rompe le scatole.
Qualche suggerimento?

robertogl 07-11-2013 11:01

Il limite di f va a uno con x che va all'infinito. Basta raccogliere sotto e sopra x.

Sent from my GT-I9505 using Tapatalk 4

zanardi84 07-11-2013 13:42

Quote:

Originariamente inviato da robertogl (Messaggio 40236765)
Il limite di f va a uno con x che va all'infinito. Basta raccogliere sotto e sopra x.

Sent from my GT-I9505 using Tapatalk 4

Sorry, ho dimenticato la x che moltiplica la parentesi nel testo.. senza sarei riuscito a risolverlo. Modifico il post sopra.

robertogl 07-11-2013 13:49

x per (e alla uno meno radice di e) fa comunque infinito. Dove ti blocchi?

Sent from my GT-I9505 using Tapatalk 4

zanardi84 07-11-2013 15:36

Quote:

Originariamente inviato da robertogl (Messaggio 40237890)
x per (e alla uno meno radice di e) fa comunque infinito. Dove ti blocchi?

Sent from my GT-I9505 using Tapatalk 4

Scusa ancora, riscrivo la funzione perchè qualcosa non quadra.

lim per x ->+ infinito di x*((e^f)- radice di e)

dove f = (x+2)/(2x+3)

Radice di e non è all'esponente.

Se confronto gli infiniti all'esponente ottengo che la f è asintotica a 1/2 per confronto di infiniti dello stesso grado. La sottrazione degli e in parentesi mi da 0, moltiplicata per x-> + infinito che è una forma di indecisione.

robertogl 07-11-2013 15:58

Quote:

Originariamente inviato da zanardi84 (Messaggio 40238586)
Scusa ancora, riscrivo la funzione perchè qualcosa non quadra.

lim per x ->+ infinito di x*((e^f)- radice di e)

dove f = (x+2)/(2x+3)

Radice di e non è all'esponente.

Se confronto gli infiniti all'esponente ottengo che la f è asintotica a 1/2 per confronto di infiniti dello stesso grado. La sottrazione degli e in parentesi mi da 0, moltiplicata per x-> + infinito che è una forma di indecisione.

Ah ok prima non c'era il 2x, o almeno non lo vedo da telefono :)
Prova a mettere x=1/y per entrare in uno di quei casi dove puoi usare Hopital, magari qualcosa esce ;)

Sent from my GT-I9505 using Tapatalk 4

CioKKoBaMBuZzo 12-11-2013 15:58

ciao a tutti

avrei una domanda su un problema di elettromagnetismo, ma la domanda riguarda la matematica che sta dietro la soluzione quindi penso sia meglio postarlo qui.

ho una superficie sferica di raggio R, una carica A posta a distanza d > R dal centro della sfera e una carica B posta a distanza c < R dal centro della sfera, tutte sullo stesso asse: centro / A / B

sono in coordinate sferiche riferite al centro della superficie sferica, e teta rappresenta l'angolo tra l'asse di cui prima (centro sfera / A / B) e il raggio vettore dal centro della sfera.

devo determinare B e c affinché il potenziale sulla superficie sferica sia nullo.
sono arrivato a dire che la condizione perchè ciò accada è:

A / sqrt( R^2 + d^2 -2*R*d*cos(teta) ) = -B / sqrt( R^2 + c^2 -2*R*c*cos(teta) )

per ogni scelta di teta tra 0 e pi greco.

a questo punto mi sono bloccato. sul libro ho visto che va avanti dicendo che ciò può accadere solo se i denominatori sono funzioni simili di teta (ho cercato la definizione di funzioni simili ma non ho trovato nulla), e in particolare deve essere:

c/R = R/d

qualcuno può aiutarmi?
grazie

Lampo89 12-11-2013 18:32

Quote:

Originariamente inviato da CioKKoBaMBuZzo (Messaggio 40263870)
ciao a tutti

avrei una domanda su un problema di elettromagnetismo, ma la domanda riguarda la matematica che sta dietro la soluzione quindi penso sia meglio postarlo qui.

ho una superficie sferica di raggio R, una carica A posta a distanza d > R dal centro della sfera e una carica B posta a distanza c < R dal centro della sfera, tutte sullo stesso asse: centro / A / B

sono in coordinate sferiche riferite al centro della superficie sferica, e teta rappresenta l'angolo tra l'asse di cui prima (centro sfera / A / B) e il raggio vettore dal centro della sfera.

devo determinare B e c affinché il potenziale sulla superficie sferica sia nullo.
sono arrivato a dire che la condizione perchè ciò accada è:

A / sqrt( R^2 + d^2 -2*R*d*cos(teta) ) = -B / sqrt( R^2 + c^2 -2*R*c*cos(teta) )

per ogni scelta di teta tra 0 e pi greco.

a questo punto mi sono bloccato. sul libro ho visto che va avanti dicendo che ciò può accadere solo se i denominatori sono funzioni simili di teta (ho cercato la definizione di funzioni simili ma non ho trovato nulla), e in particolare deve essere:

c/R = R/d

qualcuno può aiutarmi?
grazie

Secondo me fai prima in questo modo:
la relazione che trovi vale per ogni teta; in particolare per teta = 0, Pi ottieni le due equazioni:

A / sqrt( R^2 + d^2 -2*R*d ) = -B / sqrt( R^2 + c^2 -2*R*c )

A / sqrt( R^2 + d^2 +2*R*d ) = -B / sqrt( R^2 + c^2 +2*R*c )

che è sistema di due equazioni in due incognite e ti ritrovi al volo la soluzione (una non è accettabile dato che dà un c > R)

CioKKoBaMBuZzo 13-11-2013 20:01

grazie per la risposta, in effetti sembra fattibile così

ma la nozione di "funzioni simili" esiste o è una cosa intuitiva che non capisco io?

Lampo89 13-11-2013 22:29

Quote:

Originariamente inviato da CioKKoBaMBuZzo (Messaggio 40270807)
grazie per la risposta, in effetti sembra fattibile così

ma la nozione di "funzioni simili" esiste o è una cosa intuitiva che non capisco io?

non esiste una definizione, credo che sia semplicemente per dire che se hai un'uguaglianza tra due funzioni con una forma analitica simile (espressa in termini di alcune incognite che vorresti determinare), allora dovrebbe essere abbastanza semplice trovare questi parametri "confrontando le due" e facendo tornare le cose. In questo caso non mi sembra.

comunque, quell'uguaglianza lì significa che il rapporto degli argomenti delle radici deve essere una costante (lo vedi portando tutte la radici a membro dx per es, a membro sx ottieni appunto una cosa indipendente dall'angolo), perciò i due argomenti differiscono per la costante moltiplicativa (la chiamo h per essere breve) legata al rapporto fra carica/carica immagine .

R^2 + d^2 -2*R*d cos(teta)= h (R^2 + c^2 -2*R*c cos(teta) )
ottengo un polinomio in cos(teta), cosa che mi piace perché posso usare il principio di identità dei polinomi
uguaglio perciò i coeff:

d = h c
R^2 + d^2 = h (R^2 + c^2)

se risolvi sto sistema dovresti trovare la relazione che afferma il tuo libro

zanardi84 14-11-2013 10:24

Quote:

Originariamente inviato da robertogl (Messaggio 40238724)
Ah ok prima non c'era il 2x, o almeno non lo vedo da telefono :)
Prova a mettere x=1/y per entrare in uno di quei casi dove puoi usare Hopital, magari qualcosa esce ;)

Sent from my GT-I9505 using Tapatalk 4

Risolto con i criteri asintotici.

La funzione al denominatore, dopo aver razionalizzato, tende a 1 per x tendente a infinito. Posso perciò scriverla come 1+ funzione infinitesima la cui somma è la funzione che ho all'esponente.
Ottengo quindi una cosa del tipo e^(1+g(x) - e. Raccolgo e ed ottengo e(e^g(x) -1). Quello che c'è in parentesi è asintotico a g(x).

Il denominatore non crea problemi.

Entrambe le funzioni, la g(x) ottenuta e la f(x) originale al denominatore tendono a 1/2, perciò basta sostituire per ottenere il risultato di un quarto radice di e.

zanardi84 07-01-2014 15:55

Zone di definizione soluzioni cauchy
 
Ho un problema di cauchy le cui equazioni sono

y' = (y+y^2)/2
y(1) = 2

Ho risolto l'equazione e ho trovato la soluzione generale

y(x) = kx/(1-kx) dove ek = e^c con c costante.

sostituendo la condizione iniziale trovo k = 2/3, da cui y(x) = 2x/(3 - 2x)

Ora: come stabilisco qual è il più ampio intervallo su cui la soluzione del problema di cauchy è definita?

Grazie.

Vlad3210 16-01-2014 09:51

Salve a tutti!
Potete aiutarmi con questo integrale? Esiste la sua soluzione in forma chiusa?

http://operaez.net/cgi-bin/mimetex.cgi?\int\limits_0^\infty {{{\ln }^2}\left( {\frac{\lambda }{\eta } + q} \right){q^{N - 1}}{{\left( {\frac{\lambda }{\eta } + q} \right)}^{ - \left( {N + \lambda } \right)}}dq}

Grazie!


Tutti gli orari sono GMT +1. Ora sono le: 05:05.

Powered by vBulletin® Version 3.6.4
Copyright ©2000 - 2024, Jelsoft Enterprises Ltd.
Hardware Upgrade S.r.l.